Está en la página 1de 4

Name:

ENEL 441 Midterm Retest

1. (10 points) Consider this model of an antenna control system. The output is the azimuth angle of a
dish antenna, which is used to track satellites. The input is the desired angle, while the disturbance is
caused by wind blowing on the antenna.

W (s)
R(s)

V (s)

E(s)
Gc (s)

(s)

1
s(s + 1)

where the control signal is given by the equation


de(t)
+ KI
v(t) = KP e(t) + KD
dt

e( )d
0

(a) (5 points) By superposition, the output can be written as


(s) = HR (s)R(s) + HW (s)W (s)
Find the transfer functions HR (s) and HW (s)
First, find the transfer function of the controller,
Gc (s) = KD s + KP +

K I K D s2 + K P s + K I
s
s

To find HR (s), set W (s) = 0, and solve

R(s)
+

s2 KD + sKP + KI
s

1
s2 + s

(s)

(s)
R(s)

K D s2 + K P s + K I
s3 + s 2
K D s2 + K P s + K I
1+
s3 + s 2
2
KD s + K P s + K I
3
s + (1 + KD )s2 + KP s + KI

To find HW (s), set R(s) = 0, and solve


W (s)
+

s2

1
+s

(s)

(s)
W (s)

s2 KD + sKP + KI
s

Page 3 of 6

1
+s
K D s2 + K P s + K I
1+
s3 + s 2
s
3
s + (1 + KD )s2 + KP s + KI
s2

Name:

ENEL 441 Midterm Retest

(b) (5 points) Suppose that the controller was designed as follows , KI = 3, KD = 1 and KP = 1.
Let the reference input be a ramp r(t) = 2tu(t), and let the disturbance be a step w(t) = 0.1u(t).
What is lim e(t)?
t

Before using anything based on the final value theorem, we must check stability. The denominator is
P (s) = s3 + (1 + KD )s2 + KP s + KI
= s3 + 2s2 + s + 3
All coefficients are positive, so build the Routh array,
s3
s2
s
1

1
2

1
3

1
2

Since there are two sign changes in the first column, the system is unstable. Thus, the limit
does not exist.

Page 4 of 6

Name:

ENEL 441 Midterm Retest

replacements
2.PSfrag
(10 points)
Given two masses m1 and m2 , attached by a spring with spring constant k, which are free
to move on a horizontal surface that has coefficient of friction d, and where m 1 has an external force
u(t) applied
x(t)

y(t)
k

u(t)

m2

m1

we can write two differential equations, one for mass m1 and one for mass m2 , of the form
mass acceleration = friction force + spring force + external force (if any)
(a) (7 points) Starting with two such equations, take their Laplace transforms and find the block
diagram of the system with input U (s) and output Y (s) and make sure signal X(s) exists and is
labelled (hint: it will have a feedback loop).
Solution
For mass 1
PSfrag replacements

m1 x
(t) = dx(t)
+ k(y(t) x(t)) + u(t)

m1 s2 X(s) = dsX(s) + k(Y (s) X(s)) + U (s)


X(s) =

k
U (s) +
+

m1

s2

1
(kY (s) + U (s))
+ ds + k
X(s)

1
m1 s2 +ds+k

Y (s)

For mass 2

m2 y(t) = dy(t)
+ k(x(t) y(t))

m2 s2 Y (s) = dsY (s) + k(X(s) Y (s))


PSfrag replacements

Y (s) =

m2

s2

k
X(s)
+ ds + k

so the complete block diagram looks like


U (s) +

X(s)

1
m1 s2 +ds+k

+
k
k

Page 5 of 6

k
m2 s2 +ds+k

Y (s)

ENEL 441 Midterm Retest


Name:
PSfrag replacements
(b) (3 points) Consider the block diagram of a plant (containing a positive feedback loop)
U (s) +

1
s2 +s+1

1
s2 +s+1

Y (s)

+
k
1

PSfrag replacements
Find all values of KP and KD such that the following control law will result in a stable closed-loop
transfer function
u(t) = KP e(t) + KD e(t)

R(s)

E(s)

U (s)
KP s + K D s

1
(s2 +s+1)2 1

KP + K D s
Y (s)
= 4
3
R(s)
s + 2s + 3s2 + (2 + KD )s + KP
s4
s3
s2
s1
s0

:
:
:
:
:

1
2
b3
c3
KP

3
KP
2 + KD
KP

where

1
1
b3 = (2 + KD 6) = (4 KD )
2
2
and we require positive KD < 4 and
c3 =

1
(2KP (1/2)(KD 4)(KD + 2))
b3

and thus we require


(KD 4)(KD + 2) > 0

2
KD
2KD 8 > 0

(KD 1)2 10 > 0

and solving the inequality we get a positive KD must satisfy

KD < 1 + 38/2
and the restriction on KP is
2
KP < KD
/4 + KD /2 + 2

Page 6 of 6

Y (s)

También podría gustarte